Tải bản đầy đủ (.doc) (7 trang)

Bài tập về cấp của một số NGUYÊN MODULO n

Bạn đang xem bản rút gọn của tài liệu. Xem và tải ngay bản đầy đủ của tài liệu tại đây (72.66 KB, 7 trang )

Trường THPT chuyên Hoàng Văn Thụ
Sở GD&ĐT Hòa Bình
BÀI TẬP VỀ CẤP CỦA MỘT SỐ NGUYÊN MODULO n
Cho số nguyên dương n > 1 và số nguyên a nguyên tố cùng nhau với n, số k
được gọi là cấp của a modulo n nếu k là số nguyên dương nhỏ nhất để
a k ≡ 1( mod n ) .
Do tính nhỏ nhất của số nguyên dương k, nên mọi số nguyên dương h mà
h
cũng thỏa mãn a ≡ 1( mod n ) thì luôn có k | h . Và đây cũng là tính chất được sử

dụng xuyên suốt trong các bài tập về cấp của một số nguyên. Để sử dụng được tính
chất này thì ta thường kết hợp với
Định lý Phecma nhỏ: “Cho p là số nguyên tố và a là số nguyên nguyên tố cùng
p −1
nhau với p thì a ≡ 1( mod p ) ”


Định lý Ơ-le: “Cho số nguyên dương n và a là số nguyên nguyên tố cùng nhau với
ϕ ( n)
n thì a ≡ 1( mod n ) ”

Bài 1. Cho n = 1 + 2m.u , u lẻ u < 2m , m ∈ ¥ * , biết tồn tại số nguyên tố p lẻ thỏa mãn
n −1

p 2 Mn . Chứng minh rằng n là số nguyên tố.
Hướng dẫn.
Gọi q là ước nguyên tố nhỏ nhất của n. Theo giả thiết có
p

n −1
2



≡ −1(mod q) ⇒ p n−1 ≡ 1(mod q ) .

q −1
Theo định lý Phecma nhỏ có: p ≡ 1( mod q ) .

h | n − 1
Gọi h là cấp của p modulo q, ta có: 
h | q − 1
Mà p

n −1
2

≡ −1(mod q) suy ra:

n −1
/ h mà h | n − 1 suy ra h = 2m.v với v | u . Giả sử
M
2

q < n , theo tính chất của ước nguyên tố nhỏ nhất suy ra q ≤ n , mà


m
m
m
m
n = 1 + 2m.u < 22 m (vì u < 2m ) ⇒ q < 2 , lại có: 2 ≤ h = 2 .v | q − 1 < 2 . Ta thu được


điều vô lý. Vậy n = q.
Bài 2. Tìm số nguyên dương n nhỏ nhất thỏa mãn: 22013 |17 n − 1 .
Hướng dẫn.
Thực chất bài tập này đang yêu cầu chúng ta đi tìm cấp của 17 modulo 2 2013. Đặt m
= 22013 và gọi k là cấp của 17 theo modulo m, theo định lý Ơ-le có:
2012
t
17ϕ ( m ) ≡ 1(mod m) . Dẫn đến k | ϕ ( m ) = 2 , do đó k = 2 , t ∈ { 1;2;3,...,2012} .

Mặt khác ta cũng có phân tích:

(

) (

)

17 2 − 1 = ( 17 − 1) ( 17 + 1) ( 17 2 + 1) 17 2 + 1 ... 17 2 + 1 ,
t

2

t−1

do 17 ≡ 1( mod 4 ) , nên mỗi số hạng ở dạng 17 2 + 1, m ∈ { 1,2,..., t − 1} đều chia hết
m

cho 2 nhưng không chia hết cho 4, còn 17 – 1 = 16 = 2 4. Do đó 17 2 − 1 = 2t + 4.Q , Q
t


là số nguyên lẻ.
Từ 22013 |17 2 ta suy ra được t + 4 ≥ 2013 ⇒ t ≥ 2009 . Do đó để đảm bảo tính nhỏ
t

nhất của k thì k = 22009 .
2
q
Bài 3. Tìm tất cả các cặp số nguyên tố p, q sao cho p + 1| 2003 + 1 và

q 2 + 1| 2003 p + 1 .
Hướng dẫn.
Không mất tổng quát ta có thể giả sử p ≤ q .
* Ta xét trường hợp p = 2, khi đó 5 | 2003q + 1 , do đó dễ thấy q chẵn, nên q = 2. Vậy
trong trường hợp này có nghiệm p = q = 2.
2
2
* Nếu p > 2, có p lẻ và p + 1 ≡ 2 ( mod 4 ) , gọi r là một ước nguyên tố lẻ của p + 1 ,

từ giả thiết có:
2003q ≡ −1( mod r ) ⇒ 20032 q ≡ 1( mod r ) .
Gọi k là cấp của 2003 modulo r, ta có k | 2q .
q
Dễ thấy vì 2003 ≡ −1( mod r ) nên k ≠ q , suy ra k | 2 hoặc k = 2q.


2
- Nếu k | 2 dẫn đến r | 20032 − 1 = 23.3.7.11.13.167 . Mặt khác r | p + 1 nên r bằng

2 hoặc có dạng 4s + 1, mà r lẻ suy ra r = 13 . Khi đó vì 2003 ≡ 1( mod13) nên
2003q + 1 ≡ 2 ( mod13) , điều này dẫn tới mâu thuẫn.

r −1
- Nếu k = 2q, theo định lý Phecma nhỏ ta có: 2003 ≡ 1( mod r ) , suy ra 2q | r − 1

⇒ q | r − 1 , dễ thấy p 2 + 1 chẵn và mọi ước lẻ của nó đều chia q dư 1, suy ra
p 2 + 1 ≡ 2 ( mod q ) ⇒ q | p 2 − 1 = ( p − 1) ( p + 1) , nhưng với giả sử p ≤ q và do p, q là
các số nguyên tố nên q = p + 1 , do q lẻ nên p chẵn suy ra p = 2, trái với điều giả sử
p > 2.
Vậy ta chỉ tìm được một cặp nghiệm (p;q) = (2;2).
Bài 4. Tìm số nguyên dương n để 3n − 1Mn3 .
Hướng dẫn.
* Với n = 1, thỏa mãn yêu cầu bài toán.
* Với n ≥ 2 , gọi p là ước nguyên tố nhỏ nhất của n, từ giả thiết suy ra được:
3n ≡ 1( mod p ) .
p −1
Mặt khác theo định lý Phecma nhỏ ta cũng có: 3 ≡ 1( mod p ) .

h | n
Gọi h là cấp của 3 modulo p, ta có: 
như vậy ta chỉ ra được một số bé hơn
h | p − 1
p là ước của n, để p là ước nguyên tố nhỏ nhất thì h = 1. Khi đó
31 ≡ 1( mod p ) ⇒ p = 2 .
α
*
Giả sử n = 2 .t ( α , t ∈ ¥ ) , t lẻ.

( )
α

Khi đó: 3n − 1 = 32

α

2
Vì 3

( t −1)

α

+ 32

( t − 2)

t

(

)(

α

α

− 1 = 32 − 1 . 32

( t −1)

α

+ 32


( t − 2)

α

)

+ ... + 32 + 1

+ ... + 32 + 1 ≡ q ≡ 1( mod 2 ) , nên:
α

(

α −1

)(

)(
)
+ 1 là α − 1 số chẵn và ( 3 + 1) .( 3 − 1) = 8 , nên:
α −2

) (

)(

3n − 1 = 32 + 1 32 + 1 ... 32 + 1 32 + 1 32 − 1 .R (R lẻ).
α −1


α −2

Vì 32 + 1,32 + 1,...,32

1

1

0

20

0

20


3n − 1 = 2α + 2.S , (S lẻ).
Vì 3n − 1Mn3 = 23α .t 3 , suy ra α + 2 ≥ 3α ⇒ α ≤ 1 ⇒ α = 1 ⇒ n = 2t .
Giả sử n có ước lẻ thực sự, gọi ước lẻ nhỏ nhất của nó là q, và gọi k là cấp của 3
n
modulo q, theo giả thiết có: 3 ≡ 1( mod q ) và theo định lý Phecma nhỏ ta cũng có:

k | n
3q −1 ≡ 1( mod q ) . Từ đó suy ra: 
.
k | q − 1
Nếu k lẻ ta chỉ ra được một ước lẻ của n mà nhỏ hơn q, trái với cách chọn q, vậy k
chẵn, mặt khác n = 2.t (t lẻ), do đó k = 2 và n | 32 − 1 = 23 , do đó n không có ước lẻ
thực sự.

Vậy n = 2.
Bài

5.

Tìm

tất

cả

các

bộ

ba

số

nguyên

tố

(p,q,r)

thỏa

mãn:

p | q r + 1, q | r p + 1, r | p q + 1 .

(2003 USA IMO Team Selection Test)
Hướng dẫn.
* Trước hết ta chứng minh p, q, r phân biệt. Thật vậy, giả sử nếu có p = q, từ
p | q r + 1 ta suy ra điều vô lý.
* Ta chứng minh một trong ba số p, q, r phải bằng 2. Giả sử p, q, r > 2.
2r
r
Từ p | q + 1 ta suy ra q ≡ 1( mod p ) . Gọi k là cấp của q modulo p ta có k | 2r , do r

là số nguyên tố lẻ nên suy ra k = 2 hoặc k = r hoặc k = 2r.
r
r
- Nếu k = r, ta có q ≡ 1( mod p ) , mà q ≡ −1( mod p ) nên suy ra p | 2 , mâu thuẫn.
p −1
- Nếu k = 2r, theo định lý Phecma nhỏ có q ≡ 1( mod p ) , suy ra 2r | ( p − 1)

⇒ p ≡ 1( mod r ) ⇒ p q + 1 ≡ 2 ( mod r ) , kết hợp giả thiết ⇒ r | 2 , mâu thuẫn.
2
- Vậy chỉ có thể k = 2, ta có p | q − 1 = ( q − 1) ( q + 1) , do p là số nguyên tố nên

p | q − 1 hoặc p | q + 1 , nếu q ≡ 1( mod p ) , thì từ giả thiết p | q r + 1 , suy ra p | 2 ,


mâu thuẫn. Vậy p | q + 1 , vì p, q là hai số nguyên tố lớn hơn 2 nên suy ra p |

Chứng minh tương tự ta cũng có q |

q +1
.
2


r +1
p +1
, r|
. Xét số lớn nhất trong ba số p, q,
2
2

r ta thấy ngay điều nói trên là vô lý.
r
2
* Không mất tổng quát giả sử q = 2, ta có p | 2 + 1 và r | p + 1 , suy ra p, r là các số

nguyên tố lẻ. Khi đó k là cấp của 2 modulo p, theo chứng minh trên có k | 2r , do r
là số nguyên tố lẻ nên suy ra k = 2 hoặc k = r hoặc k = 2r.
p −1
Nếu k = r hoặc k = 2r tức là r | k , theo định lý Phecma nhỏ có 2 ≡ 1( mod p ) , suy
2
2
ra k | p − 1 ⇒ r | p − 1 ⇒ p + 1 ≡ 2 ( mod r ) mà r | p + 1 , trái với r là số nguyên tố

lẻ.
2
Do đó k = 2, khi đó p | 2 − 1 = 3 ⇒ p = 3 và r | 32 + 1 = 10 ⇒ r = 5 .

Vậy chỉ tìm được duy nhất một bộ số thỏa mãn là (2;3;5).
Bài 6. Chứng minh rằng với mọi số nguyên dương n, 3n − 2n không chia hết cho n.
Hướng dẫn.
Giả sử tồn tại số nguyên dương n nào đó để 3n − 2n Mn . Ta sẽ chỉ ra điều vô lý. Gọi p
là ước nguyên tố nhỏ nhất của n, từ 3n − 2n Mn suy ra p ≥ 5 . Do p lẻ nên tồn tại số

nguyên

dương

a

để

2a ≡ 1( mod p ) ,

p +1

a =
÷.
2 


3n ≡ 2n ( mod p )

Từ

⇒ ( 3a ) ≡ 1( mod p ) .
n

Gọi k là cấp của 3a modulo p, ta có k | n , mặt khác theo định lý Phecma nhỏ ta có

( 3a )

p −1


≡ 1( mod p ) , nên k | p − 1 . Từ cách chọn p là ước nhỏ nhất của n suy ra k =

1. Vậy ta có 3a ≡ 1( mod p ) và 2a ≡ 1( mod p ) , suy ra a ≡ 0 ( mod p ) , điều này mâu
thuẫn với 2a ≡ 1( mod p ) .
Bài 7. Tìm tất cả các cặp số nguyên dương m, n thỏa mãn n |1 + m3 + m 2.3 .
n

n

(Bulgarian IMO Team Selection Test)


Hướng dẫn.
Với n = 1, là trường hợp tầm thường, mọi m nguyên dương đều thỏa mãn.
Với n > 1, dễ thấy (m,n) = 1. Gọi k là cấp của m modulo n. Từ n |1 + m3 + m 2.3 suy
n

n

ra n | m3 − 1 , vì vậy k | 3n+1 ⇒ k = 3t , t ≤ n + 1 .
n+1

ϕ ( n)
Theo định lý Ơ-le có m ≡ 1( mod n ) , suy ra k | ϕ ( n ) .

Nếu k = 3n +1 , từ k | ϕ ( n ) ⇒ k < n , ta thu được điều mâu thuẫn.
Vậy

t ≤ n ⇒ k = 3t | 3n


⇒ n | m3 − 1
n

⇒ m 2.3 + m3 + 1 ≡ 3 ( mod n ) ,
n

n



n |1 + m3 + m 2.3 ⇒ n = 3 và m ≡ 1( mod3) .
n

n

x7 − 1
= y 5 − 1.
Bài 8. Tìm tất cả các nghiệm nguyên của phương trình:
x −1
(Dự tuyển IMO 2006)
Hướng dẫn.
Giả sử phương trình có cặp nghiệm nguyên (x;y), và gọi p là một ước nguyên tố bất
x7 − 1
= x 6 + x 5 + ... + x + 1 , khi đó xảy ra hai trường hợp:
kỳ của
x −1
- Nếu x ≡ 1( mod p ) khi đó

x7 − 1
= x 6 + x 5 + ... + x + 1 ≡ 7 ( mod p ) , và khi đó p = 7.

x −1

7
- Nếu x ≡ 1( mod p ) , gọi k là cấp của x modulo 7, do p | x − 1 nên k | 7 , do

x ≡ 1( mod p ) , ta tìm được k = 7. Dễ thấy (x,p) = 1, nên theo định lý Phecma nhỏ ta
p −1
có x ≡ 1( mod p ) , do đó k = 7 | p − 1 ⇒ p ≡ 1( mod 7 ) .

Vậy trong cả hai trường hợp ta luôn có p ≡ 0;1( mod 7 ) , với p là một ước nguyên tố
x7 − 1
x7 − 1
của
. Điều này dẫn đến mọi ước dương của
cũng có số dư là 0 hoặc 1
x −1
x −1
khi chia cho 7.
5
4
3
2
Mặt khác y − 1 = ( y − 1) ( y + y + y + y + 1) , theo nhận xét trên ta rút ra được

y − 1 ≡ 0,1( mod 7 ) và y 4 + y 3 + y 2 + y + 1 ≡ 0,1( mod 7 )


4
3
2

Điều này mâu thuẫn vì khi đó y ≡ 1,2 ( mod 7 ) ⇒ y + y + y + y + 1 ≡ 5,3 ( mod 7 ) .

Vậy phương trình vô nghiệm.



×